What is Eletromagnetism: Definition and 14 Discussions

No Wikipedia entry exists for this tag
  1. C

    I Preventing parasitics for wires?

    This is related to a question that I asked here: https://physics.stackexchange.com/questions/650709/one-way-magnetic-shielding. Please take a look at the check-marked answer. There is a material that can act as a one-way magnetic/electric shield. My question is if this material were to be used...
  2. G

    A Bremsstrahlung single photon or spectrum

    Non-relativistic Bremsstrahlung is discussed classically in Rybicki “Radiative Processes in Astrophysics” where Larmor’s formula is used to find the power radiated in a collision between an electron and a Coulomb field. The Fourier transform of the pulse allows for a description of the pulse in...
  3. Moara

    Eletromagnetism: Copper Plate on a Spring Oscillating in a Magnetic Field

    Tried to find the resultant force, but I can't see how the magnetic field affects. I used Faraday's law to find the the diferece of potentials in the plate Wich should be B.d.v, where v is the vertical velocity of plate, but there were not given the resistance or resistivity to relate with the...
  4. migueldbg

    Potential from point charge at distance ##l## from conducting sphere

    After looking around a bit, I found that, considering the polar axis to be along the direction of the point charge as suggested by the exercise, the following Legendre polynomial expansion is true: $$\begin{equation}\frac{1}{|\mathbf{r} - \mathbf{r'}|} = \sum_{n=0}^\infty...
  5. K

    Strength of magnetic field at the end of a wire wrapped nail.

    V=I*R 6v=I*(0.6+0.9)ohms I=4amp B=100*(uo)(2N)(I)/L * 1/2 I think since the wire is double wrapped, we multiply the equation by 2, but since we are looking for the magnetic field at the end of the wire we also have to multiply the equation by 1/2 I=4A, uo= 4pi*10^-7 2N/L turns per unit...
  6. Celso

    Magnetic field due to a straight wire

    Can someone help me to understand some of these steps? Why can we write ##\frac{1}{R} = \frac{sin(\phi)}{R}## and ##y = - cot(g\phi)##?
  7. S

    Explaining Temperature Increase with Antenna and Tissue

    Hi; Let's imagine that I have an antenna and a biological tissue. The antenna emits the frequency of 1e7 Hz and I have a temperature increase x. When the antenna emits the frequency of 3e7 and 6e7 Hz the temperature increase is the same. How can I explain?
  8. carllacan

    Transformer under an external magnetic field.

    What happens to the current through a transformer windings if the whole system is under a uniform magnetic field in the direction of the coils and with frequency equal to that of the AC in current? I think the current induced in the primary solenoid should generate a field that would make up...
  9. F

    Deriving Magnostatics equations from steady currents

    I've always heard that maxwell's equations contains essentially all of eletromagnetic theory. However, there's one thing I'm having trouble doing for myself: deriving the magnestatics equations from the maxwell's equations. Of course: it's clear that if you put ∂[t]E=∂[t]B=0 (partial derivative...
  10. Y

    How do we eliminate the existance of luminiferous aether?

    According to Maxwell's equation, the speed of light, ##C_0 = \frac{1}{\sqrt{\epsilon_0\mu_0}}##, is a constant regarding to some form of medium, called luminiferous aether. Shortly after the death of Maxwell, Michelson-Morley Experiment shows that the speed of light is constant regarding to the...
  11. A

    Why Does the Mirror Charge Method Double the Potential Energy?

    Hi everybody, The situation is the classic one: a point charge q+ in a distance d above a conductor plane grounded: The conductor is grounded so V = 0, for z = 0. Also, far away from the system (x2 + y2 + z2 >> d) V --> 0 The argument to replace it for a q- charge seems perfect to me. What I...
  12. G

    How do the magnetic fields produce emfs?

    I should add context to this question that what I know so far of the emf is that it is defined as ##\mathscr E=\oint f \cdot d\ell## where ##f## is the sum of the forces on the charges. In the textbook I'm reading it gives an example of where a person pulls a rectangular wire with a velocity...
  13. tsuwal

    Triple integral in eletromagnetism: prove force doesnt depend radius.

    Homework Statement This problem may be dull, I know, but maybe there is a hidden math trick that i don't know of. This picture sums up the problem. So, you should prove by simplifing the integral that F^e, the eletric force applied between two spheres, onde with a charge q_1 and the...
  14. F

    Minimize Impedance - (eletromagnetism)

    OK, I think I am doing this question right, but I'm not exactly sure. The question is as follows: For an RLC circuit with a resistance of 16k\ohm , a capacitance of 8.0\mu F and an inductance of 38.0H what frequency is needed to minimize the impedance? Well impedance is give by: Z =...
Back
Top